identify the 3D shape :)thank you

Identify The 3D Shape :)thank You

Answers

Answer 1
Answer: Triangular prism

If you folded the figure up, you would have a prism where the parallel bases are right triangles. Each lateral face is a rectangle.

It might help to imagine a room where the floor and ceiling are triangles (they are identical or congruent triangles). Each wall of this room is one of the rectangles shown.


Related Questions

Hi, could someone help me solve this. so the question says to find the area of the shaded part (in black) , in terms of pie (π). the length of the square is 12 cm. the radius of the circle is 6cm. i came with the answer of (144-36π)/4. is this ok? below is the picture of the question. ​

Answers

Answer:

yes but can be simplified

Step-by-step explanation:

area of shaded part = ( area of square - area of circle ) / 4

= [tex]\frac{12^2-\pi (6)^2}{4}[/tex]

= [tex]\frac{144-36\pi }{4}[/tex]

= [tex]\frac{144}{4}[/tex] - [tex]\frac{36\pi }{4}[/tex]

= 36 - 9π

Here is a table of values for y = f(x).
Х
-2 -1 0 1 2 3
4.
5
6
f(x) 5
6 7 8 9 10 11 12 13
Mark the statements that are true.

Answers

Step-by-step explanation:

the true answers are:

A. f(-1)=6

D. the domain for f(x) is the set

{-2,-1,0,1,2,3,4,5,6}

Which expression is equivalent to 4p^-4 10q -3? Assume ​

Answers

Step-by-step explanation:

Derive an expression for the equivalent width in a saturated line. Assume a Voigt profile, with the difference in optical depth between the center of the line and the wings being ~104. The wings of the line can be ignored. Define a frequency x1 = (v1 − v0)/ΔvD, where the optical depth τv = 1. Inside of x1 the line is fully saturated, and outside x1 the line is optically thin. Show that the equivalent width is

Note that the equivalent width is practically insensitive to the number density of absorbing material.

PLEASE HELP!

Determine which of the following lists is in order from smallest to largest.

1. -3,131,0, (-3)^2
2. (-3)^2,-3,0, |3|
3. -3,0,|3|, (-3)^2
4. 0,-3,|3|, (-3)^2

Answers

Answer:

3. -3,0,|3|, (-3)^2

Step-by-step explanation:

Answer:

answer would be option 3

Step-by-step explanation:

help this helps

what does $42,690e(0.03)×(20) equal​

Answers

Answer:

77,786.251

Step-by-step explanation:

Answer:

77,786.251 is the answer that equals $42,690e(0.03)×(20)

Step-by-step explanation:

hey mate hope this help you...!

(Right angle Trigonometry) Please help me solve and explain!

Answers

Answer:

31.3

Step-by-step explanation:

Reference angle given = 56.1°

Hypotenuse = 56.1

Adjacent side = x = ?

We would apply CAH, which is:

Cos 56.1° = Adj/Hyp

Cos 56.1° = x/56.1

56.1 × Cos 56.1° = x

31.2895006 = x

x = 31.3 (approximated to the nearest tenth)

find the area of the following figures: ​

Answers

Answer:

Area = 156 square cm

Step-by-step explanation:

What is the midpoint of the line segment connecting the points (-4, 5) and (4,7)?

Answers

Answer:

(0,6)

Step-by-step explanation:

Answer:

(0,6)

Step-by-step explanation:

We need to find the midpoint of the line segment having end points (-4,5) and (4,7). So, here we can use the Midpoint Formula :-

Points :- (-4,5) and (4,7).

Using Midpoint Formula ,

[tex]\rm\implies Midpoint =\bigg(\dfrac{x_1+x_2}{2},\dfrac{y_1+y_2}{2}\bigg)[/tex]

Substitute the respective values ,

[tex]\rm\implies Midpoint =\bigg(\dfrac{-4+4}{2},\dfrac{7+5}{2}\bigg)[/tex]

Simplify the numbers ,

[tex]\rm\implies Midpoint =\bigg(\dfrac{0}{2},\dfrac{12}{2}\bigg)[/tex]

This equals to ,

[tex]\rm\implies \boxed{\blue{\rm Midpoint =\bigg( 0,6\bigg)}}[/tex]

Hence the midpoint is (0,6) .

Use the distributive property to write the next step in simplifying th
numerical expression
7(2 + 7)

A. 7• 2+7•7
B. 7• 2+7
C. (7+2) • (7 + 7)
D. 7+2 • 7+ 7

Answers

Answer:

A)

Step-by-step explanation:

(View attachment)

a) Write ordered pairs.
b) Write the domain and range.
c) Why isn't the relation a function?
d) Which ordered pair should be removed to make the relation a function?

Answers

Answer:

in a relationship that maps elements from one set (the inputs) into elements from another set (the outputs), the usual notation for the ordered pairs is:

(x, y), where x is the input and y is the output.

In this case, the point where the arrow starts is the input, and where the arrow ends is the output.

a)

The ordered pairs are:

(28, 93)

(17, 126)

(52, 187)

(34, 108)

(34, 187)

b) The domain is the set of the inputs, in this case the domain is the set where all the arrows start, then the domain is:

{17, 28, 34, 52}

And the range is the set of the outputs, in this case the range is:

{93, 108, 126, 187}

c) A function is a relationship where the elements from the domain, the inputs, can be mapped into only one element from the range.

In this case, we can see that the input {34} is being mapped into two different outputs, then this is not a function.

d) We can remove one of the two ordered pairs where the input is {34},

So for example, we could remove:

(34, 108)

And then the relation would be a function.

A.54 pie cm^3

B.72 pie cm^3

C.126 pie cm^3

D.378 pie cm^3

Answers

Answer:  54pi  cm^3   (choice A)

==========================================================

Explanation:

The radius of each sphere is r = 3

The volume of one sphere is

V = (4/3)*pi*r^3

V = (4/3)*pi*3^3

V = 36pi

That's the volume of one sphere.

Three spheres take up 3*36pi = 108pi cm^3 of space.

---------------------------

The radius of the cylinder is also r = 3, since each tennis ball fits perfectly in the container.

The height is h = 18 because we have each ball with a diameter 6, which leads to the three of them stacking to 3*6 = 18.

The volume of the cylinder is...

V = pi*r^2*h

V = pi*3^2*18

V = 162pi

-------------------------

Subtract the volume of the cylinder and the combined volume of the spheres:  162pi - 108pi = (162-108)pi = 54pi

This is the exact volume of empty space inside the can.

This points to choice A as the final answer

HELP!!!! NUMBER 2

2. Find the distance between these two points.
First Point
Second Point
Distance
(5, - 2)
(8,6)
(5,6)
(-3,6)
(-3,6)
(5,-2)

Answers

Answer:

1. 8

2 . 11

[tex]3 . \sqrt{128} \ or \ 8 \sqrt2[/tex]

Step-by-step explanation:

[tex]Distance = \sqrt{(x_2 -x_1)^2 + (y_2 - y_1)^2 }[/tex]

1 . ( 5 , - 2 ) and ( 5 , 6 )

  [tex]Distance = \sqrt{(5 -5)^2 + ( 6-(-2))^2} = \sqrt{ 0 + 8^2 } = \sqrt{64} = 8[/tex]

2 . ( 8 , 6 ) and ( - 3 , 6 )

  [tex]Distance = \sqrt{( -3 -8)^2 + ( 6 - 6)^2 } = \sqrt{ (-11)^2 + 0 } = \sqrt { 121 } = 11[/tex]

3. ( 5 , - 2 ) and ( -3 , 6 )

  [tex]Distance = \sqrt{(-3 - 5)^2 + ( 6 --2)^2} = \sqrt{(-8)^2 + ( 8)^2} = \sqrt{ 64 + 64 } = \sqrt{128}[/tex]

   

          [tex][ \ \sqrt{128} = \sqrt{ 2 \times 64} = \sqrt{ 2 \times 8^2 } = 8 \sqrt{2} \ ][/tex]

what is 5.73 as a mixed number​

Answers

Answer:

[tex]5 \ \frac{73}{100} [/tex]

Step-by-step explanation:

Convert the decimal number to a fraction by placing the decimal number over a power of ten. Since there are

2 numbers to the right of the decimal point, place the decimal number over 10^2 (100). Next, add the whole number to the left of the decimal.

Answer:

5 73/100

Step-by-step explanation:

5.73 = 573

100

= 573

100

as a fraction

To convert the decimal 5.73 to a fraction, just follow these steps:

Step 1: Write down the number as a fraction of one:

5.73 = 5.73

1

Step 2: Multiply both top and bottom by 10 for every number after the decimal point:

As we have 2 numbers after the decimal point, we multiply both numerator and denominator by 100. So,

5.73

1

= (5.73 × 100)

(1 × 100)

= 573

100

.

(This fraction is alread reduced,

As the numerator is greater than the denominator, we have an IMPROPER fraction, so we can also express it as a MIXED NUMBER, thus 573

100

is also equal to 5 73/100

when expressed as a mixed number.

3 3/4 × 2 2/9 please
Help ‍♀️‍♀️‍♀️

Answers

[tex]\implies {\blue {\boxed {\boxed {\purple {\sf { \: 8 \frac{1}{3}\:(or) \:8.333}}}}}}[/tex]

[tex]\sf \bf {\boxed {\mathbb {Step-by-step\:explanation:}}}[/tex]

[tex]3 \frac{3}{4} \times 2 \frac{2}{9} [/tex]

➺[tex] \: \frac{15}{4} \times \frac{20}{9} [/tex]

➺[tex] \: \frac{300}{36} [/tex]

➺[tex] \: \frac{25}{3} [/tex]

➺[tex] \: 8 \frac{1}{3} [/tex]

➺[tex] \: 8.333[/tex]

[tex]\large\mathfrak{{\pmb{\underline{\pink{Mystique35 }}{\orange{❦}}}}}[/tex]

Sketch the region enclosed by the given curves. Decide whether to integrate with respect to x or y. Draw a typical approximating rectangle and label its height and width.4x+y^2=12, x=yThen find the area S of the region.

Answers

Answer:

The Area of the Enclosed Region Is 64/3.

Step-by-step explanation:

As Given in Question

We have, Curve 4x+[tex]Y^{2}[/tex]=12

& X=Y

Solution.

4Y+[tex]Y^{2}[/tex]=12   (X=Y)

[tex]Y^{2}[/tex]+4Y-12=0

[tex]Y^{2}[/tex]+6Y-2Y-12=0

Y(Y+6)-2(Y+6)=0

(Y-2)*(Y+6)=0

Y=2 & -6  (X=Y)

Now at (2,2) & (-6,-6) both curves intersect each other.

The Area Of Enclosed Region is    [tex]\int\limits^2_{-6} [(3-Y^{2}/4 )-Y] \, dy[/tex]

by Solving This Equation we get Area of Region = 64/3 .    

this equation Solution & Curve Diagram please see In Attachment .  

9x5
pls help meeeeeeeeee

Answers

Answer:

45

hope this helps

Answer:

45

Step-by-step explanation:

9x5=45

Area of a trapezoid
Find the area of this trapezoid. Be sure to include the correct unit in your answer.
6 cm
0
10 cm
8 cm
12 cm

Answers

Answer:

h

Step-by-step explanation:

h


Anne invested $1000 in an account with a 3% annual interest rate. She made no deposits or
withdrawals on the account for 2 years. If interest was compounded annually, which equation
represents the balance in the account after the 2 years?

Answers

for this problem you find 3 percent of $1000 to calculate the first year of interest which is 30 and then add it to your balance to get $1030 then you find 3 percent of that which is 30.9 and add it to your balance to get a total of $1060.9

What number must be added to both the numerator and denominator of the fraction 2/5 to get 7/8?
a) 13
b) 15
c) 17
d) 19​

Answers

The answer is D.
Adding 19 to both 2 & 5 in 2/5 will equal 21/24. 21/24 can then be simplified to 7/8.

ASAP there are three marbles in a bag. One is red and two are black. What is the probability of picking a black marble first, putting it back in the bag and then picking a black marble? Use the following probability to find the answer.

Answers

Answer:

[tex] \frac{4}{9} [/tex]

Step-by-step explanation:

[tex]p = \frac{favorable \: outcomes}{total \: outcomes} = \frac{4}{9} [/tex]

Answer:  Choice D)  4/9

=============================================================

Explanation:

The probability you get a black marble on the first selection is 2/3 since we have 2 black marbles out of 2+1 = 3 total.

We put the marble back and then we have 2/3 as the probability of selecting another black marble on the second try. Nothing has changed because we put the marble back. That means the events are independent.

So we get (2/3)*(2/3) = 4/9 as the probability of selecting 2 black marbles in a row (with replacement).

474 ( ) 548 42. - 8 how do I arrive at this answer.

Answers

9514 1404 393

Answer:

  (64x^28)/(5y^22)

Step-by-step explanation:

The applicable rules of exponents are ...

  (a^b)^c = a^(bc)

  (a^b)(a^c) = a^(b+c)

  (a^b)/(a^c) = a^(b-c)

__

First, eliminate the outside exponent on the left factor. Then use the above rules to combine factors with the same base.

  [tex]\displaystyle\left(\frac{4x^4}{5y^6}\right)^5\cdot\left(\frac{5^4y^8}{4^2x^{-8}}\right)=\frac{(4^5x^{20})(5^4y^8)}{(5^5y^{30})(4^2x^{-8})}=4^{5-2}5^{4-5}x^{20-(-8)}y^{8-30}\\\\=4^35^{-1}x^{28}y^{-22}=\boxed{\frac{64x^{28}}{5y^{22}}}[/tex]

the sum of two consecutive numbers is 2x+3. What are the numbers

Answers

Answer: 2 and 3

Step-by-step explanation:

its numbers

differentiate loge(x/x^2+7)​

Answers

Answer:

1+1=11 2+2=22 ok na yan kuya or ate

smallest to biggest pls

Answers

Step-by-step explanation:

45.399,45.454,45.545,45.933

The amount of increase from the cost to the selling price is called the?​

Answers

Answer:

profit

hope it is helpful to you

Step-by-step explanation:

Markup is the percentage amount by which the cost of a product is increased to arrive at the selling price.

Find the area for me pls

Answers

this figure can be divided into three parts one is rectangle other one is semicircle and the third one is one fourth of the circle .so let's find the area of each figure one by one. For the rectangle 12×8 =96

for semicircle that is on the top it has the radius 6 which is a half of 12

so area of the semicircle is

[tex] \frac{1}{2} \pi \: r {}^{2} \\ \frac{1}{2} \times 3.14 \times 6 {}^{2} \\ 3.14 \times 18 \\ 56.52[/tex]

no that's fine. 80 of the 1/4 of the other Circle

[tex] \frac{1}{4} \pi {}^{2} \\ \frac{1}{4} 3.14 \times 8 {}^{2} \\ 3.14 \times 16 \\ 50.24[/tex]

add all these areas

96+56.52+50.24 =202.76

please help meeeee!!​

Answers

Step-by-step explanation:

[tex]\begin{aligned} -5x+4y &= 3\\\\ x&=2y-15 \end{aligned}[/tex]

please solve fast please ​

Answers

Answer:

3/6 =1/25/6

pls give me brainliest plzzzzz

Chung has 6 trucks and 5 cars in his toy box. Brian has 4 trucks and 5 cars in his toy box.

Which is the correct comparison of their ratios of trucks to cars?
StartFraction 6 Over 4 EndFraction less-than StartFraction 5 Over 5 EndFraction
StartFraction 6 Over 4 EndFraction greater-than StartFraction 5 Over 5 EndFraction
StartFraction 6 Over 5 EndFraction less-than StartFraction 4 Over 5 EndFraction
StartFraction 6 Over 5 EndFraction greater-than StartFraction 4 Over 5 EndFraction

Answers

Given:

Chung has 6 trucks and 5 cars in his toy box.

Brian has 4 trucks and 5 cars in his toy box.

To find:

The correct comparison of their ratios of trucks to cars.

Solution:

The ratio of trucks to cars is defined as:

[tex]\text{Ratio}=\dfrac{\text{Number of trucks}}{\text{Number of cars}}[/tex]

Chung has 6 trucks and 5 cars in his toy box. So, the ratio of trucks to cars is:

[tex]\text{Ratio}=\dfrac{6}{5}[/tex]

Brian has 4 trucks and 5 cars in his toy box.

[tex]\text{Ratio}=\dfrac{4}{5}[/tex]

We know that,

[tex]6>4[/tex]

[tex]\dfrac{6}{5}>\dfrac{4}{5}[/tex]

Therefore, the correct option is D.

Answer:

what the guy above me said

Step-by-step explanation:

so yeah he is right points

Consider the graph below: Point T(-2; 3) is a point on the Cartesian Plane such that B is the angle of inclination of OT. T(-2;3) у х 2.1 Calculate the following without the use of a calculator: a) tanſ b) 13 sin B.cosB (2)​

Answers

Answer:

(a) - 3/2

(b) - 78/25

Step-by-step explanation:

According to the trigonometry, the tangent of any angle is the ratio of rise to the run of the right angle triangle .

The sine of an angle is the ratio of rise to the hypotenuse of the right angle triangle.

The cosine of an angle is the ratio of run to the hypotenuse of the right angle triangle.

(a)

[tex]tan\beta = \frac{3}{-2} = \frac{-3}{2}[/tex]

(b)

[tex]13 sin\beta cos \beta = 13\times \frac{3}{\sqrt{3^2+2^2}}\times\frac{-2}{\sqrt{3^2+2^2}}\\\\13 sin\beta cos\beta = \frac{- 78}{25}[/tex]

Other Questions
what do you mean by bone The fracture strength of tempered glass averages 14 (measured in thousands of pounds per square inch) and has standard deviation 2. (a) What is the probability that the average fracture strength of 100 randomly selected pieces of this glass exceeds 14.2 Which of the following accurately describes an interaction between two religions during the period of 600 BCE to 600 CE?A. Hindu traditions became central to ZoroastrianismB. Buddism provided the foundation for HinduismC. Christianity grew out of the Jewish faithD. Islam developed as an extension of Buddhism Trong mt lp hc c 50 sinh vin. Hi c bao nhiu cch bu ra mt ban cn s lp gm 3 ngi: 1 lp trng, 1 lp ph, 1 b th v khng kim nhim chc v. The length of second hand of clock is 14cm, an ant sits on the top of second hand. find the followingi) speed of antii) distance covered by ant in 150 secondsiii) displacement in 150 seconds Find the area of the figureWillGive Brainlist Which scientific claim can be made? Songbirds numbers will increase due to the effects of climate change increasing global Temperatures Which of the following is an example of procedural law? A. The order in which the steps of a trial take place B. The specific civil rights listed in the U.S. Constitution C. What an individual is allowed to do in public locations D. The amount of a certain chemical that can be present in manufacturing waste Which process involves glucose reacting with oxygen to produce energy, carbon dioxide, and water? Which of the following items requires an internalcooking temperature of 165 F?a. Pork chopsb. Injected meats c. Raw Eggsd. poultry My sisters house is 1 2/4 times as high as my house. My house is 5 feet high. How high is my sisters house? Analyze the diagram below and complete the statement that follows.The perimeter of the square isA. 42B. 60C. 110.25D. 112.5 Trevor heard a burglar entering through a living room window.He grinned as he picked up his gun. Crouching behind the sofa in his darkened home,he ambushed and killed the intruder with several well placed shots.He then added another notch in his trusty side-arm.Trevor most probably:____________ a. has exercised his constitutional right of self-defense. b. has acted legally,because the shooting took place inside his home. c. has acted legally if,but only if,the burglar was armed with a gun. d. is guilty of a homicide,or at least voluntary manslaughter. can someone help me???? 3.42x16.5 show your work plz Which three factors determine the formality of a discussion?O opinion, audience, and timeO topic, audience, and purposeO facts, purpose, and locationO topic, time, and location **PLEASE HELP**The frequency table below represents the 30 best battling averages for a semi pro baseball league. Which ranges of battling averages were least common among the players what is a reflection of a story X+ 5If m(x) =x-1 and n(x) = x-3, which function has the same domain as (mon)(x)?X+5O (x)=1111o h(x)=X-111O (X)=X-411Oh(x) =X-3 Neglecting air resistance and the weight of the propellant, determine the work done in propelling a five-ton satellite to a height of (a) 100 miles above Earth and (b) 300 miles above Earth.